Difference between revisions of "1998 AHSME Problems/Problem 22"

(Solution 1)
 
(10 intermediate revisions by the same user not shown)
Line 8: Line 8:
 
\qquad\mathrm{(D)}\ 2
 
\qquad\mathrm{(D)}\ 2
 
\qquad\mathrm{(E)}\ 10</math>
 
\qquad\mathrm{(E)}\ 10</math>
== Solution ==
 
 
=== Solution 1 ===
 
=== Solution 1 ===
 
By the change-of-base formula,  
 
By the change-of-base formula,  
 
<cmath>\log_{k} 100! = \frac{\log 100!}{\log k}</cmath>   
 
<cmath>\log_{k} 100! = \frac{\log 100!}{\log k}</cmath>   
Thus (you might recognize this [[identity]] directly)
+
Thus (because if <math>x=y</math>, then <math>\frac{1}{x}=\frac{1}{y}</math>)
 
<cmath>\frac{1}{\log_k 100!} = \frac{\log k}{\log 100!}</cmath>
 
<cmath>\frac{1}{\log_k 100!} = \frac{\log k}{\log 100!}</cmath>
 
Thus the sum is  
 
Thus the sum is  
<cmath>\left(\frac{1}{\log 100!}\right)(\log 1 + \log 2 + \cdots + \log 100) = \frac{1}{\log 100!} \cdot \log 100! = 1 \Rightarrow \mathrm{(C)}</cmath>
+
<cmath>\left(\frac{1}{\log 100!}\right)(\log 2 + \log 3 + \cdots + \log 100) = \frac{1}{\log 100!} \cdot \log 100! = 1 \Rightarrow \mathrm{(C)}</cmath>
  
 
=== Solution 2 ===
 
=== Solution 2 ===

Latest revision as of 10:41, 22 April 2016

Problem

What is the value of the expression \[\frac{1}{\log_2 100!} + \frac{1}{\log_3 100!} + \frac{1}{\log_4 100!} + \cdots + \frac{1}{\log_{100} 100!}?\]

$\mathrm{(A)}\ 0.01 \qquad\mathrm{(B)}\ 0.1  \qquad\mathrm{(C)}\ 1 \qquad\mathrm{(D)}\ 2 \qquad\mathrm{(E)}\ 10$

Solution 1

By the change-of-base formula, \[\log_{k} 100! = \frac{\log 100!}{\log k}\] Thus (because if $x=y$, then $\frac{1}{x}=\frac{1}{y}$) \[\frac{1}{\log_k 100!} = \frac{\log k}{\log 100!}\] Thus the sum is \[\left(\frac{1}{\log 100!}\right)(\log 2 + \log 3 + \cdots + \log 100) = \frac{1}{\log 100!} \cdot \log 100! = 1 \Rightarrow \mathrm{(C)}\]

Solution 2

Since $1=\log_{k} k$,

\[\frac{1}{\log_{k}100!}=\frac{\log_{k}k}{\log_{k}100!}=\log_{100!} k\]

We add:

\[\log_{100!} 1 +\log_{100!} 2 +\log_{100!} 3 +\cdots + \log_{100!} 100=\log_{100!}100!=1 \Rightarrow \mathrm{(C)}\]

See also

1998 AHSME (ProblemsAnswer KeyResources)
Preceded by
Problem 21
Followed by
Problem 23
1 2 3 4 5 6 7 8 9 10 11 12 13 14 15 16 17 18 19 20 21 22 23 24 25 26 27 28 29 30
All AHSME Problems and Solutions

The problems on this page are copyrighted by the Mathematical Association of America's American Mathematics Competitions. AMC logo.png